User Avatar
peteryzhu843
Joined
Apr 2025
Subscription
Free
User Avatar
peteryzhu843
Thursday, Oct 30 2014

Thanks for all the detailed responses. What got me on this problem was Powerscore LR bible states "most strongly supports" as a way the author words a "must be true" question stem. This really restricted my interpretations of the answer choices. I definitely agree that the stimulus does not support (B) but I felt that (C) was refutable with evidence from the stimulus and therefore picked (B) over (C).

User Avatar

Wednesday, May 29 2013

peteryzhu843

Merits of Retaking PTs?

I read in several places that retaking PTs is recommended. I'm wondering what are the merits of this? How should we treat the second run through differently? Perhaps we should go through the test as fast as we can to test our gut instincts rather than carefully thinking through each one?

User Avatar
peteryzhu843
Wednesday, Oct 29 2014

Thanks for the reply! Could you elaborate a little more? I interpreted the clause that supported (C) as rather saying that the people who are allergic to cats aren't allergic to the same proteins and therefore a cat may or may not cause an allergic reaction. I find (C) possible and not necessarily true.

Maybe the root of the issue is that I interpreted the question stem "most strongly supported by the information above" as "must be true".

User Avatar
peteryzhu843
Monday, Oct 27 2014

anyone?

User Avatar
peteryzhu843
Tuesday, Aug 26 2014

ah i see my mistake now. Thanks!

User Avatar

Tuesday, Aug 26 2014

peteryzhu843

PT30.S2.Q20 - justified public funding

http://classic.7sage.com/lsat_explanations/lsat-30-section-2-question-20/

I found this question especially tricky because of what I perceive to be a logic gap in the stimulus. The critics argument can be boiled down to (Justified public funding) -> (indicated that public will benefit). The stimulus then goes on to say if this relationship holds true, then we would not be seeing the public support for this project, suggesting

!(indicated that public will benefit) -> !(justified public funding for this project) -> !(public support)

Is this the right interpretation? I have trouble accepting the second part which asserts that justification of public funding is necessary for public support of a project. I don't see where this is verified in the stimulus.

User Avatar
peteryzhu843
Tuesday, Nov 26 2013

I see my error now. Thanks!

http://classic.7sage.com/lsat_explanations/lsat-56-section-3-question-20/

The argument first states a general claim about people taking Vitamin C is, on average, healthier and then proceeds to show how he arrived at that conclusion by citing a study about heart disease. I picked (B) over the correct answer (D) without hesitation. The question stem asks "which one of the following, if true, would most weaken the argument in the newspaper article?"

I felt (D), the claim that Vitamin C supplements tend to reduce one's resistance to certain common infectious diseases weakens the conclusion but does not weaken the argument---one that derives the benefits of Vitamin C from the fact that studies have shown it lowers the risk of heart disease.

While (B) isn't an ideal answer, I felt it fit more because it weakens the argument rather than the premise or the conclusion by implying that Vitamin C may not be responsible for the reduction in the risk of heart disease. Furthermore, the question stem asks for "most weaken" rather than "which of the following would weaken" which I read as the answer does not necessarily have to fit perfectly but only needs to be slightly better than the best alternative.

Could someone explain why (D) would be superior to (B) given the question stem?

Thanks!

User Avatar
peteryzhu843
Thursday, Sep 25 2014

I'm planning on taking the December LSAT but if you're open to meeting up, just shoot me a message. I live out in Tysons Corner but am free to meet up in NoVA or DC on the weekends.

User Avatar
peteryzhu843
Tuesday, Oct 22 2013

Hey thanks for the response! I realize my logical fault is confusing trying to strengthen a relation with trying to prove it.

User Avatar
peteryzhu843
Monday, Oct 21 2013

Thanks all for the detailed responses

@, I probably should clarify the if and only if signals a definition here and not A (-) B relationship. I'm saying "A causes B" if and only if three conditions are satisfied.

You mention that if we introduce chemical T in the water, chemical T causes the effect of fish developing physical deformation. What are the conditions under which you can conclude the cause-effect relationship in that example?

What prompted my question here is PT45 S1 #12. It asks to weaken the conclusion that a causation relationship does not exist, which I interpreted (perhaps falsely) it means to strengthen the argument a causation relationship does exist. The answer gets at condition 2 showing B does not occur in the absence of A.

I guess the question comes down to: when can we conclude a causal relation exists? I often see LSAT questions that ask you to refute an argument that establishes these relations but it doesn't escape my imagination it can also ask you to make an argument for establishing them.

User Avatar

Monday, Oct 21 2013

peteryzhu843

Defining a causal relationship

Prior to the LSAT I've never really considered why A causes B. I'm very unsure of my definition of a causal relationship and would like to put it out there for everyone to critique.

A causes B if and only if

1. B occurs whenever A occurs (A -> B)

2. B does not occur if A does not occur (B -> A)

3. A occurs temporally before B

For example, say we know that toxin T is strongly correlated with fish dying in large numbers. Say we also know that in the absence of toxin T, fish populations are always high. Lastly, we know that toxin T is always released prior to any die off of fish. Can we conclude toxin T _causes_ the decrease in fish populations?

User Avatar

Tuesday, Oct 21 2014

peteryzhu843

PT68.S2.Q10 - people who are allergic to cats

http://classic.7sage.com/lsat_explanations/lsat-68-section-2-question-10/

My interpretation of the stimulus is that people who are allergic to cats react to certain proteins within the cat's skin and saliva though which particular proteins vary between people. Every cat is capable of causing some sort of allergic reaction in someone though a particular cat may cause a reaction in some but not others. I disliked all of the answers and ended up choosing (B) because it was the one I found the hardest to refute.

Could someone explain why it's (C)? I feel that the stimulus suggests it's possible all cats secrete the same proteins and that it's the allergy sufferers that vary in which protein they react thereby directly refuting (C). It's supported by the clause "...which particular proteins are responsible, however, varies from allergy sufferer to allergy sufferer..."

User Avatar
peteryzhu843
Thursday, Nov 20 2014

Thanks Socal and Jdawg for the explanations. I understand now where the substitution is happening. Jdawg's explanation made it especially clear. I feel like given game time situations in under a wide range of conditions I would not have interpreted this answer choice with that substitution but hopefully going forward I won't make the same mistake twice. Thanks again

User Avatar
peteryzhu843
Wednesday, Jun 19 2013

I have this inkling the longer the question is, stimulus and questions combined (Parallel Reasoning, Parallel Flaw both easy and hard ones), the probability the answer is A/B goes up and D/E goes down. This is just cause I think the LSAT has a built in reward system for those who really know the concept over those who rely on eliminating answer choices.

Practically speaking, this just means on longer questions start reading from A rather than skip around like I have a bad habit of.

User Avatar
peteryzhu843
Wednesday, Nov 19 2014

Understanding the stimulus was much less the issue than understanding the answer choice here. Quoting answer choice (C): "... the risk of fatal injury to the motorcyclist thus incurred is acceptable ..." It seems to me the answer choice is talking about the risk of fatal injury itself is acceptable while both you and I agree that the stimulus is pointing out the activities that pose risks to life are acceptable.

It just seems to me (C) is completely irrelevant to the stimulus. Could you speak more to this point?

User Avatar
peteryzhu843
Wednesday, Jun 19 2013

I see flaw questions as the exact same as assumption questions. I ask myself "what do I need to assume to make this argument work." Then I look for the answer choice that basically says that assumption can't be made.

User Avatar
peteryzhu843
Tuesday, Nov 18 2014

So (risk is acceptable) -> (activity that poses that risk is acceptable)? I don't know if we can accept that, especially since the stimulus doesn't state it directly. There are probably a lot of additional factors that must be considered before an activity can be deemed acceptable (cost, legality, etc).

User Avatar
peteryzhu843
Tuesday, Nov 18 2014

Thanks for the reply. My problem with (C) is that it states the RISK itself is acceptable while the stimulus only states whether the ACTIVITY that poses that risk is acceptable. I had trouble picking (A) precisely for the reason you mentioned but I felt that was easier to accept than what appears to be a blatant incompatibility between (C) and the stimulus. Could you elaborate more on this point?

User Avatar
peteryzhu843
Friday, Oct 18 2013

I use to really struggle with concentration as well. I started forcing myself to read, almost anything really, for at least an hour a day and that dramatically improved my stamina. Also, be cognizant of what you eat. Diets high in fat tend to dampen your focus. Also, don't take PTs on a full stomach. With regards to sleep, I read somewhere you do your best when you've had about 1 hr less sleep than you're naturally accustomed to. For most people that means 6 hours or so. The slight sleep deprivation causes a small adrenaline kick which is perfect for cognitive tasks.

User Avatar
peteryzhu843
Friday, Oct 18 2013

There's something I started doing for precisely those LR questions where I'm gambling between two answer choices and I think it has helped me develop an intuition. I write down these questions---stimulus, question stem, both answer choices---by hand as many as three to five times over (parallel questions included). The idea is to ingrain into me the habit of recognizing the key pivot words to the point I can unconsciously conclude an answer choice is wrong without having to consciously understand why. Unfortunately, it's more difficult to conclude a choice is right.

http://classic.7sage.com/lsat_explanations/lsat-60-section-3-question-18/

I had a lot of trouble with this question picking between A/C (reasons outlined below) and am hoping someone could point out the error in my logic.

The stimulus states activities that pose a risk are acceptable if and only if they satisfy any of two conditions

1) the net benefit outweigh the risk

2) the person bearing the risk does so voluntarily

Starting with the answer I chose first

(C) A motorcyclist rides without a helmet and the RISK is acceptable because he's doing so voluntarily. At first glance this seemed to definitely fit as an application of the principle stated in the stimulus. The motorcyclist is choosing to ride without a helmet. However, the answer states the risk is acceptable while the principle is talking about whether the activity that poses this risk is acceptable. One might argue that an acceptable activity necessarily implies the risk is acceptable. However, here all we're told is the necessary condition is satisfied and we can't conclude anything about the sufficient condition.

Therefore, I changed my answer to (A) though I had a lot of reservations:

(A) a salesperson voluntarily and therefore this activity is acceptable. This didn't seem to fit the stimulus too well. How do we know the risk the salesperson, by not replacing his old car with a new one with new safety features, is imposing a risk on his life? I felt (C) was blatantly wrong (stating that the risk is acceptable rather than the activity) while (A) seemed a little less wrong since it didn't seem too large a leap of faith to assume his old car may pose a risk on his life. After all, isn't driving always posing a risk on your life?

Between these two, I ended up picking (A) which happened to be the wrong answer of the two. Could anyone point out what I'm missing here?

Thanks in advance,

Peter

User Avatar
peteryzhu843
Monday, Aug 11 2014

Hey, I might be a little late to the party but I live right at Tyson's Corner. I took the LSAT last Dec but cancelled and I put everything off for a year. I'm getting back into it now and plan on writing the Dec one again. I average 173-175 and my strengths traditionally have been LG though my LR has been improving. RC is by far my weakest. I work a 9-8 day job but am willing to meet up 9pm on weekdays (if you're close by Tyson's) and mostly free on weekends. Let me know if you're still interested.

User Avatar
peteryzhu843
Friday, Jun 07 2013

For unless, "A will occur unless B occurs", I think of it as B is necessary for stopping A (or B is necessary for not A). That would be (not) A -> B

ex. Spurs will sweep unless Heats wins a game. In other words, Heats must win a game to stop Spurs from sweeping.

(not) Spurs sweep series -> Heats wins a game

User Avatar
peteryzhu843
Thursday, Jun 06 2013

ahh I've gotten at least one wrong on every law passage. I probably should just take the hint.

User Avatar
peteryzhu843
Friday, Dec 05 2014

ah, that really makes the question a lot easier. This was the question that separated me from breaking my target score grrr

User Avatar
peteryzhu843
Tuesday, Jun 04 2013

Addressing PrepTest 9, Section 2, Question #23:

I'm going to make an assumption here that might be faulty but I think based on the stimulus, should be safe to make. a and b are disjoint and x and y are disjoint. That is a (-) not b and x (-) not y. If you're poor, you can't be rich, if you're honest, you can't be dishonest and vice versa.

we know if you're poor then you're honest

p -> h

We want to conclude if you're rich then you're dishonest

not p -> not h

We can only arrive at this if

p (-) h (if and only if)

In other words, if we must assume the opposite: h -> p

It really helps me understand the concepts of assumption questions by looking at it formally. Could someone check my logic?

So say we have a premise (B->C) and we have the conclusion (A->C) and say we have the two question stems

Question stem 1: Which one of the following, if assumed, ALLOWS the conclusion to be properly drawn?

Question stem 2: Which one of the following assumptions is REQUIRED for the conclusion to be properly drawn?

For question stem 1, we're looking for a sufficient assumption. Is that just saying something like

___ (and) (B->C) -> (A->C)

so in this case (A->B) would be the obvious sufficient assumption that fills the blank?

For question stem 2, we're looking for the necessary assumption. Is that saying

(B->C) (and) (A->C) -> ___

where the necessary assumption fills in the blank? Something about this doesn't seem quite right.

User Avatar
peteryzhu843
Wednesday, Sep 03 2014

What I found the hardest about LR is often its confusing arrangement of words. What helped me improve significantly was writing down the stimulus, my answer choice and the correct answer choice of every problem I've had issues with by hand several times over (unfortunately this includes parallel reasoning questions as well). This helped me internalize the language so that at least understanding each problem becomes second nature.

User Avatar
peteryzhu843
Tuesday, Sep 03 2013

Hi everyone, coming back from a 3 month break on studying and just getting back into it (planning on writing the Dec test). I'm getting 172-174 on most of my PTs (goal is 175+) and finding it hard to get motivated again by myself. I'm in the DC area and work a day job so will be studying mostly on weekends. Lemme know by message if you're interested.

User Avatar
peteryzhu843
Friday, Jan 02 2015

@ curve was -12

User Avatar
peteryzhu843
Friday, Jan 02 2015

Unless I had a faulty test, the perfume one has to be the real one since it was my only RC section

Hi all,

In the December LSAT, I had an experimental LR section so only one RC section (section 4). I remember clearly two of the articles were on perfume and stealing thunder. In the LSAC's Disclosure Book uploaded RC section, neither of these articles were included. Anyone else notice this issue?

Peter

User Avatar

Saturday, Jun 01 2013

peteryzhu843

PT56.S2.Q17 - personal right society's welfare

How I read the stimulus was: say we let Q be the statement "create virtuous people"

Glen: I believe Q is most important because P (an alternative) is not desirable

Sara: But Q is more dangerous than P

I can't seem to get past why answer choice C is wrong. Glen's closing sentence states he endorses law's primary role to create virtuous citizens implying there is negligible danger in making Q the most important. On the other hand Sara counters by saying this is more dangerous than being overprotective of individual rights, thereby implying she believes there is an inherent danger in the government deciding what constitutes virtuous behavior. Wouldn't this point be something they directly disagree about?

So E summarizes Glen's argument, but Sara simply disagrees by stating Q is more dangerous than an alternative and seems to imply Q may not be the most important. But I feel that the level of interpolation to go from Sara's argument (Q being more dangerous) to Sara believing Q is not the most important is the same as that for C.

http://classic.7sage.com/lsat_explanations/lsat-56-section-2-question-17/

Confirm action

Are you sure?